Областная олимпиада по математике, 2023 год, 9 класс


Для положительных чисел $a,b,c,a_1,b_1,c_1$ таких, что $2 \le \frac{x}{x_1} \le 18$ $(x=a,b,c)$, докажите неравенство $$(a^2+b^2+c^2)(a_1^2+b_1^2+c_1^2)\le \frac{25}{9}(aa_1+bb_1+cc_1)^2.$$
посмотреть в олимпиаде

Комментарий/решение:

  4
2023-02-25 15:23:03.0 #

Ну кароч тут типа заметим, что $x-2x_1 \geq 0$ и $18x_1-x \geq 0$. Перемножим оба неравенства и получим, что $20xx_1 \geq x^2+36x_1^2$. Тогда $20aa_1+20bb_1+20cc_1 \geq a^2+36a_1^2+b^2+36b_1^2+c^2+36c_1^2$. По нер-ву Коши для правой стороны мы получим $a^2+36a_1^2+b^2+36b_1^2+c^2+36c_1^2 \geq 2*6*\sqrt{(a^2+b^2+c^2)(a+b+c)}$.

Тогда $\frac{5}{3}(aa_1+bb_1+cc_1) \geq \sqrt{(a^2+b^2+c^2)(a+b+c)}$. Возведем обе стороны в квадрат и получим исходное неравенство, которое надо было доказать.

  5
2023-02-26 15:26:05.0 #

В истории сайта matol первый раз вижу как пользователь вот так оформляет задач: "Ну кароч тут типа заметим,"

  1
2023-02-26 16:16:06.0 #

мне нрав

  1
2023-02-28 11:42:00.0 #

спс

  0
2023-12-18 16:28:53.0 #

при каких значениях достигается точка равенства ?

  0
2023-12-19 10:41:29.0 #

при каких-то положительных

  0
2023-12-19 20:45:46.0 #

назар говорил не напишешь точку равенства то -2 балла

  0
2023-12-21 15:20:27.0 #

c=15/4

a=4/18

b=15/18

  0
2024-01-05 17:02:53.0 #

Давайте обобщим задачу. Пусть $ \dfrac{a_k}{b_k} \in [ \alpha, \beta], \alpha > 0, k = 1,2,...,n$

Тогда:

$\left({ \beta – \dfrac{a_k}{b_k}} \right) \cdot \left({ \dfrac{a_k}{b_k} – \alpha} \right) \geq 0$

Следовательно:

${a_k}^2 + \alpha \beta {b_k}^2 \leq ( \alpha + \beta) a_k b_k$ $k = 1, 2, ..., n$

Тогда

$\sum \limits_{k=1}^{n}{{a_k}^2} + \alpha \beta \sum \limits_{k=1}^{n}{{b_k}^2} \leq \left( \alpha + \beta \right) \sum \limits_{k=1}^{n}{a_k b_k}$

Из неравенства $AM \geq GM$ получаем

$ \left( \sum \limits_{k=1}^{n}{{a_k}^2} \right)^ {\dfrac{1}{2}} \left( \alpha \beta \sum \limits_{k=1}^{n}{{b_k}^2} \right)^{ \dfrac{1}{2}} \leq \dfrac{1}{2} \left( \sum \limits_{k=1}^{n}{{a_k}^2} + \alpha \beta \sum \limits_{k=1}^{n}{{b_k}^2} \right)$

Значит

$ \left( \sum \limits_{k=1}^{n}{{a_k}^2} \right)^{ \dfrac{1}{2}} \left( \alpha \beta \sum \limits_{k=1}^{n}{{b_k}^2} \right) \dfrac{1}{2} \leq \dfrac{1}{2} \left( \alpha + \beta \right) \sum \limits_{k=1}^{n}{a_kb_k}$